Solving Linear Dependence of a Matrix

Click For Summary
The discussion revolves around determining the linear dependence of a given matrix through row reduction. The matrix was reduced to show that it is not a basis since the last row is all zeros, indicating linear dependence. Participants highlight the importance of recording row operations to identify the relationship among the vectors. A non-trivial linear relation can be derived from the homogeneous system AX=0, which reflects a linear combination of the matrix's columns. Clarification is sought regarding whether the focus is on rows or columns, as interpretations may differ.
mr_coffee
Messages
1,613
Reaction score
1
Hello everyone, I'm finishing up some matrices review and im' confused on this question i have the matrix:
-1 -3 -1 2
5 13 3 -8
3 10 9 -8
1 4 7 -4

I row reduced got this:
1 0 0 3/5
0 1 0 -4/5
0 0 1 -1/5
0 0 0 0

So you can see that this isn't a basis due to column 5 not being 0 0 0 1, but what does this mean the questions says:
If they are linearly dependent, determine a non-trivial linear relation - (a non-trivial relation is three numbers which are not all three zero.) Otherwise, if the vectors are linearly independent, enter 0's for the coefficients, since that relationship always holds.
?A + ?B + ?C + ?D = 0.
I tried 1 1 1 3/5
1 1 1 0, i tried actually 14 times, all of them are wrong hah, any help?>
 
Physics news on Phys.org
Did you record the row operations that you used? Because they tell you the relationship, the reduced row of zeroes is a linear combination of the 4 rows, just recall what the combination is.
 
I used a Ti-83 calculator to find the row reduction, he said to use them for these problems!
 
You know how to solve a homogeneous system? If A is your matrix of column vectors, a non-trivial solution to the homogeneous system AX=0 will give you a non-trivial linear relation between your vectors.

AX is just a linear combination of the columns of A after all.

edit-are you concerned with a linear combination of the rows or the columns of your matrix? matt and i answered the different interpretations (in that order).
 
Last edited:
Question: A clock's minute hand has length 4 and its hour hand has length 3. What is the distance between the tips at the moment when it is increasing most rapidly?(Putnam Exam Question) Answer: Making assumption that both the hands moves at constant angular velocities, the answer is ## \sqrt{7} .## But don't you think this assumption is somewhat doubtful and wrong?

Similar threads

  • · Replies 2 ·
Replies
2
Views
2K
  • · Replies 2 ·
Replies
2
Views
1K
Replies
9
Views
2K
  • · Replies 2 ·
Replies
2
Views
2K
Replies
2
Views
2K
  • · Replies 3 ·
Replies
3
Views
1K
Replies
4
Views
2K
  • · Replies 4 ·
Replies
4
Views
1K
  • · Replies 9 ·
Replies
9
Views
2K
  • · Replies 3 ·
Replies
3
Views
3K